Easy Tutorial
For Competitive Exams

Aptitude Arithmetic & Geometric Progression Test Yourself

2551.Evaluate: $\dfrac{(2.39)^2-(1.61)^2}{3.39-1.61}$
2
4
6
8
Explanation:

Given Expression =$\dfrac{a^2- b^2}{a - b} $=$ \dfrac{(a + b)(a - b)}{(a - b)} $= a + b = 2.39 + 1.61 = 4.

2552.What decimal of an hour is a second ?
.0025
.0256
.00027
.000126
Explanation:

Required decimal =$ \dfrac{1}{60 \times 60} $=$ \dfrac{1}{3600} $= .00027

2553.The value of $\dfrac{(0.96)^3-(0.1)^3}{(0.96)^2+0.096-(0.1)^2}$ is
0.86
0.95
0.97
1.06
Explanation:

Given expression

=$ \dfrac{(0.96)^3- (0.1)^3}{(0.96)^2+ (0.96 \times 0.1) + (0.1)^2} $

= $ \left(\dfrac{a^3- b^3}{a^2 + ab + b^2} \right) $

= $\left( a - b \right)$= (0.96 - 0.1)= 0.86

2554.The value of $\dfrac{0.1 \times 0.1 \times 0.1 + 0.02 \times 0.02 \times 0.02}{0.2 \times 0.2 \times 0.2 + 0.04 \times 0.04 \times 0.04}$ is
0.0125
0.125
0.25
0.5
Explanation:

Given Expression = $\dfrac{(0.1)^3+(0.02)^3}{2^3[(0.1)^3+(0.02)^3]}$ = $ \dfrac{1}{8}$= 0.125

2555.If 2994 ÷ 14.5 = 172, then 29.94 ÷ 1.45 = ?
0.172
1.72
17.2
172
Explanation:

=$\dfrac{29.94}{1.45}$=$\dfrac{299.4}{14.5}$

=$\left(\dfrac{2994}{14.5}\times\dfrac{1}{10}\right)$[ Here, Substitute 172 in the place of 2994/14.5 ]

=$\dfrac{172}{10}$

=17.2

2576.The correct expression of 6.$\overline{46}$ in the fractional form is:
$ \dfrac{646}{99} $
$ \dfrac{64640}{1000} $
$ \dfrac{640}{100} $
$ \dfrac{640}{99} $
Explanation:

6.$\overline{46}$ = 6 + 0.$\overline{46}$ = 6 +$ \dfrac{46}{99} $ =$ \dfrac{594 + 46}{99} $ =$ \dfrac{640}{99} $.

2577.The fraction 101 $\dfrac{27}{100000}$ in decimal for is:
.01027
.10127
101.00027
101.000027
Explanation:

101$ \dfrac{27}{100000} $= 101 +$ \dfrac{27}{100000} $= 101 + .00027 = 101.00027

2578.$\dfrac{0.0203 \times 2.92}{0.0073 \times14.5 \times 0.7}$=?
0.8
1.45
2.40
3.25
Explanation:

$ \dfrac{0.0203 \times 2.92}{0.0073 \times 14.5 \times 0.7} $=$ \dfrac{203 \times 292}{73 \times 145 \times 7} $=$ \dfrac{4}{5} $= 0.8

2579.4.036 divided by 0.04 gives :
1.009
10.09
100.9
None of these
Explanation:

$\dfrac{4.036}{0.04}$=$ \dfrac{403.6}{4} $= 100.9

2580.3.87 - 2.59 = ?
1.20
1.2
1.27
1.28
Explanation:

3.87 - 2.59 = 3 + 0.87 - 2 + 0.59

=$ \left(3 +\dfrac{87}{99} \right) $-$ \left(2 +\dfrac{59}{99} \right) $

= 1 +$ \left(\dfrac{87}{99} -\dfrac{59}{99} \right) $

= 1 +$ \dfrac{28}{99} $

= 1.28.

Share with Friends